Théorème de Darboux

Bonsoir

Soit $f$ une fonction dérivable sur $I$.
1/ Soit $a,b \in I$ tels que $f'(a)<0$ et $f'(b)>0$. Démontrer qu'il existe $c$ compris entre $a$ et $b$ tel que $f'(c)=0$.
2/ Démonter que $f'(I)$ est un intervalle.


Je n'arrive pas à traiter la question 1. Pourriez-vous me donner une indication ?
Merci d'avance.
«1

Réponses

  • A quelle condition pour une fonction $f$ définie et dérivable sur un intervalle $[a ; b]$ existe-t-il $c \in ]a ; b[$ tel que $f'(c)=0$?
  • D'après le théorème de Rolle, la condition est : $f(a)=f(b)$.
  • Il y a un autre cas moins particulier qui te permet d'être sûr qu'il existe $c \in ]a ; b[$ tel que $f'(c)=0$. Tu sais ça depuis la terminale au moins (voire la 1ère).
  • penser aux extremums de la fonction
  • J'ai revu mon cours.

    Si $a$ est un point de l'intérieur de $I$, si $f$ est dérivable en $a$ et si la fonction $f$ admet un extremum relatif en $a$ alors $f'(a)=0$.
  • Considérons $a<b$

    Si je considère $g=f_{|[a,b]}$ elle est continue sur un segment $[a,b]$ elle admet donc un maximum et un minimum.

    Mais comment savoir si je dois étudier le minimum ou le maximum ?
  • Peu importe mais a et b peuvent ils être un extremum?

    D'autre part une remarque le théorème de Rolle donne une condition suffisante et pas une condition nécessaire
  • Oui je sais que le théorème de Rolle donne une fonction suffisante.

    D'après le théorème que j'ai cité, si l'extremum est atteint en $c$ je dois avoir $c \ne a$ et $c \ne b$ car l'intérieur de $[a,b]$ est $]a,b[$.

    Mais je n'arrive pas à démontrer que $c \ne a$ et $c \ne b$.
  • Au voisinage de $a$ on a : $\dfrac{f(x)-f(a)}{x-a} < 0$ et au voisinage de $b$ on a $\dfrac{f(x)-f(b)}{x-a} > 0$

    Mais ensuite je bloque.
  • a (resp b) peut il être un minimum?
  • $f'(a)<0$ donc $f(a)>f(x)$ pour tout $x>a$ au voisinage de $a$. Donc $a$ ne peut être un minimum.
    De même pour $b$.
    Par conséquent $f$ atteint son minimum en un $c$ intérieur au segment $]a;b[$
  • D'accord merci j'ai compris. Je réfléchis à la question 2 et mettrai mon raisonnement.
  • Encore une remarque : si $a>b$ on étudie un maximum...
  • Ok merci.

    Montrons que $f'(I)$ est un intervalle c'est à dire que $\forall y \in f'(I) \ \forall z \in f'(I) \ [y,z] \in f'(I)$

    Soit $y,z \in f'(I)$ ainsi il existe $a,b \in f'(I)$ tel que $y=f'(a)$ et $z=f'(b)$. Supposons $y < z$

    Soit $f'(a) < u < f'(b)$ alors en posant $g(x)=f(x)-ux$ on a $g'(x)=f'(x)-u$

    Et $g'(a)=f'(a)-u<0$ puis $g'(b)=f'(b)-u>0$ ainsi d'après la question 1 il existe $c \in ]a,b[ \ g'(c)=0$

    Il existe $c \in ]a,b[$ tel que $f'(c)=u$ donc $u \in f'(I)$

    L'inclusion est démontrée, $f'(I)$ est un intervalle.

    J'ai une petite question concernant la proposition suivante :

    Si $a$ est un point de l'intérieur de $I$, si $f$ est dérivable en $a$ et si la fonction $f$ admet un extremum relatif en $a$ alors $f'(a)=0$

    Pourquoi $a$ doit forcément être dans l'intérieur de $I$ ?
  • Parce que la dérivée est définie à l’intérieur, sinon on parle de dérivée à droite ou à gauche.
  • Question qui n'a rien à voir avec Oshine ni son exercice !

    Vous aimez ça, vous ?
    Soit $y,z \in f'(I)$ ainsi il existe $a,b \in f'(I)$
    On dirait que la ponctuation "virgule" est élément de l'ensemble !

    Cela demande vraiment du temps d'écrire : Soit $(y,z) \in f'(I)^2$ ainsi il existe $(a,b) \in f'(I)^2$ ?

    Je sais ! On le voit de plus en plus souvent mais cela me semble une transgression inacceptable de la règle : " ne jamais considérer les symboles mathématiques (surtout ceux de la théorie des ensembles) comme des signes sténographiques" !

    Ou alors faut arrêter de faire la guerre à ceux qui écrivent "pour qu'$\exists$ ...".
  • @rakam, très juste. À ceci près que $(a,b)\in I^2$ plutôt...:-D
    Autre chose pour OShine, dixième ligne : justifie que $]a;b[$ est dans $I$.

    Pour la dernière question OShine, c’est une condition suffisante, pas nécessaire.
    B&B, je ne comprends pas ta réponse.
  • Moi j'ai compris le message de Boole et Bill :)

    @Amatoué

    $]a,b[ \subset [a,b]$. Or $I$ est un intervalle donc $[a,b] \subset I$ d'où $]a,b[ \subset [a,b] \subset I$.
  • On pouvait faire directement le 2. sans s'emm.. à sav0ir si $f$ est extrémale en $a,b$.

    Soit $(a,b)\in I^2,\;u,v$ définies sur $I$ par
    $$u(x)=\begin{cases}\dfrac{f(x)-f(a)}{x-a}\quad&\text{si $x\neq a$}\\f'(a)&\text{si $x=a$}\end{cases};
    \qquad v(x)=\begin{cases}\dfrac{f(x)-f(b)}{x-b}\quad&\text{si $x\neq b$}\\f'(b)&\text{si $x=b$}\end{cases}$$
    $u,v$ sont continues sur $I$ et $u(b)=v(a)$.

    Soit $\lambda$ strictement entre $f'(a),\;f'(b)$.
    Alors $(u(a)-\lambda)(v(b)-\lambda)<0$
    donc $(u(a)-\lambda)(v(b)-\lambda)(u(b)-\lambda)^2\leq0$
    donc $[(u(a)-\lambda)(u(b)-\lambda)]\,[(v(b)-\lambda)(v(a)-\lambda)]\leq0$
    Un des crochets (par exemple le premier) est négatif et il existe $k$ entre $a,b$ tel que $u(k)=\lambda$.
    La formule des accroissements finis permet de conclure.
  • @Rakam

    Tour ça m'a l'air bien compliqué ::o

    Je ne comprends pas le passage de la ligne 7 à la ligne 8.

    Je ne comprends pas comment obtenir le $u(k)=\lambda$

    Je ne comprends pas le rapport avec la formule des accroissements finis.
  • @OShine : Il n'y a rien de compliqué si on est capable de suivre un raisonnement simple ligne à ligne.

    Ligne 7 à ligne 8 : si $ab \leq 0$ alors $a \leq 0$ ou $b \leq 0$. Ensuite l'inégalité $(u(a)-\lambda)(u(b)-\lambda)$ veut dire que $u - \lambda$ change de signe entre $a$ et $b$, donc par le théorème des valeurs intermédiaires, il existe $k \in ]a, b[$ tel que $u(k)=\lambda$. Enfin, le théorème des accroissements finis dit exactement qu'il existe $c$ entre $a$ et $k$ tel que $f'(c) = u(k) = \lambda$.
  • Je ne comprends pas le passage de $(u(a)-\lambda)(v(b)-\lambda)(u(b)-\lambda)^2\leq0 $ à $[(u(a)-\lambda)(u(b)-\lambda)]\,[(v(b)-\lambda)(v (a)-\lambda)]\leq0$ ni pourquoi on multiplie par $(u(b)-\lambda)^2$

    Je ne comprends pas pourquoi l'inégalité $(u(a)-\lambda)(u(b)-\lambda)$ veut dire que $u - \lambda$ change de signe entre $a$ et $b$

    Et c'est quoi le rapport entre le théorème des accroissements finis et $u(k)=\lambda$

    Le théorème des accroissements finis c'est $f(b)-f(a)=f'(c)(b-a)$
  • Je ne comprends rien à vos solution c'est du chinois.

    Tant pis je reste sur la méthode de l'exercice de mon livre que j'ai comprise.
  • Bon, exercices à savoir faire sur la dérivation parfaitement.

    1) Montrer que $f$ telle que
    $f(x) = \exp(-1/x^2)$ si $x$ non nul et $f(0) = 0$ est infiniment dérivable.

    2) Dérivabilité à droite en 0 de $f(x) = \cos(\sqrt{x})$.

    3) Soit $f$ dérivable et continue sur $[a,b]$ telle que $f(a) = f(b) = 0$. Montrer qu'il existe $c \in\, ]a,b[$ tel que $f'(c) + 2019f(c) = 0$ (On pourra penser à utiliser $e^{\lambda x}$ pour $\lambda$ bien choisi.
  • @Noobey
    Merci mais je commence à me décourager de ne rien comprendre aux solutions des intervenants.

    Même avec l'aide de Poirot je ne comprends pas la solution de Rakam, suis-je nul ? :(
  • @Oshine : en voici une autre (mais ce n'est pas très différent fondamentalement). Quitte à traiter quatre cas similaires tu peux supposer que $f(a)>f(b)$ et $f'(a)>0$. Voici ensuite comment procéder.

    (1) Faire un dessin.

    (2) On observe que comme $f'(a)>0$, pour $x$ proche de $a$ on a $f(x)>f(a)$. Notons $x_0$ un tel point. Alors comme $f(x_0)>f(a)>f(b)$ et que $f$ est continue le théorème des valeurs intermédiaires te donne un point $x_1>a$ tel que $f(x_1)=f(a)$.

    (3) Théorème de Rolle sur $[a,x_1]$ et c'est fini.
  • Merci @Jacky super clair ! Mais c'est uniquement pour traiter la question 1 non ?

    Ici on est dans le cas $f'(a)<0$ et $f'(b)>0$ mais votre méthode est adaptable à tous les cas :)
  • @Oshine, au sujet de l'idée de @rakam
    Elle est très astucieuse mais on peut mener la démonstration autrement:

    $u$ et $v$ étant continues, $u(I)$ et $v(I)$ sont des intervalles qui ont un point commun donc $J=u(I) \cup v(I)$ est un intervalle, et en plus $f'(a)$ et $f'(b)$ sont dedans.

    Soit $d$ strictement entre $f'(a)$ et $f'(b)$, alors $d \in J$ donc par exemple il existe $\alpha \in I$ tel que $u(\alpha)=d$ donc $\dfrac {f(\alpha)-f(a)}{\alpha-a}=d$

    En appliquant Rolle à la fonction $f$, on en déduit qu'il existe $c \in I$ tel que $f'(c)=\dfrac {f(\alpha)-f(a)}{\alpha-a}=d$.

    $f'(I)$ est donc bien un intervalle.
  • OShine est en train de péter les plombs parce qu’il pense qu’il existe une méthode, une « démarche » systématique qui permettrait de rédiger des démonstrations.
    Une démarche qui permettrait de trouver des idées astucieuses, comme celle de @rakam. Donc répondre à sa détresse en lui sortant d’autres démonstrations qu’il s’empressera bien sûr de juger « non pédagogiques »(au passage, on ne sait toujours pas ce qu’il entend par là) ne lui sera pas d’un grand intérêt.
    Il faut tout simplement lui dire, comme le disait @Foys il y a peu, que cette méthode marche bien, un point c’est tout. On parvient à trouver des démonstrations astucieuses, alors on est bon, on y parvient moins alors on est moins bon. Lire et même comprendre(après combien d’efforts?) une démonstration (qui viendrait de je ne sais quelle « démarche ») n’est pas extraordinaire puisqu’une machine sait bien le faire(comprendre=valider c’est tout!).
    Après, à force de « valider » des démonstrations @OShine, ton cerveau devrait être capable de réinvestir ou d’adapter certaines idées dans d’autres exercices, mais pour une classe limitée d’exercices correspondants.
    Les mathématiques sont une discipline difficile.
  • Les deux seules démonstrations (dans ce fil et que je connaisse) du théorème de Darboux (Il y en a peut-être une troisième dans le Gourdon, à vérifier) sont: passer par un extremum à l'intérieur de l'intervalle et construire la fonction introduite par @rakam (vu dans le livre d'exercices du RDO, idée que je trouve très jolie...). Je trouverais dommage que @Oshine passe à côté de cette deuxième démo alors qu'il est plongé là-dedans. Elle vaut le détour.
  • Un article du Monthly pour résumer la chose.
  • Franchement je sympathise avec Oshine, pourquoi se passer d'une méthode claire ? Ici on comprend, c'est une solution qu'un étudiant peut trouver.
  • Après une lecture superficielle des messages précédents, voici mon commentaire sur la belle démonstration rappelée par rakam. Ses fonctions $u$ et $v$ sont continues sur $I=[a,b]$ et donc les ensembles-images $u(I)$ et $v(I)$ sont des intervalles (et même des segments mais peu importe). Comme dit rakam, on a $u(b)=v(a)$, ce qui implique que $u(I)\cap v(I)\neq \varnothing $. Par suite $J=u(I)\cup v(I)$ est un intervalle et comme cet intervalle contient $f'(a)<0$ et $f'(b)>0$, il contient $0$. Il existe donc $\alpha \in I$ et $\beta \in I$ tels que $\alpha <\beta$ et $\displaystyle \frac {f(\beta)-f(\alpha)}{\beta -\alpha}=0$, soit : $f(\alpha)=f(\beta)$, et le théorème de Rolle permet de conclure.
    Moi aussi je connaissais cette démonstration depuis longtemps, et il me semble qu'elle est à peu près celle du Monthly donnée par Eric, qu'il en soit remercié. Mais on se demande pourquoi cette revue, en 2004, qualifie cette démonstration de « nouvelle ».
    Bonne soirée.
    Fr. Ch.
  • Mais la démonstration que je trouve la plus naturelle, c'est de considérer le minimum de la fonction continue $f$ sur le segment $I=[a,b]$, qui ne peut être atteint en $a$ sans quoi $f'(a)=\lim_{x \rightarrow a , x>a} \frac {f(x)-f(a)}{x-a} \ge 0$, ni en $b$ sans quoi $f'(b)=\lim_{x \rightarrow b, x<b} \frac {f(b)-f(x)}{b-x} \le 0$, ce qui n'est pas. Ce minimum est donc atteint en un point $c \in ]a,b[$, et alors on applique la démonstration du théorème de Rolle.
    Bonne soirée.
    Fr. Ch.
  • Prolongement.
    Appelons fonction de Darboux une fonction définie sur un intervalle de $\mathbb R$, à valeurs réelles, qui satisfait à la propriété des valeurs intermédiaires.
    Toute fonction continue est une fonction de Darboux.
    Le théorème de Darboux en question ici dit que toute fonction dérivée est une fonction de Darboux.
    Questions : la fonction somme (resp. produit) de deux fonctions de Darboux est-elle nécessairement une fonction de Darboux ?
    Bonne soirée.
    Fr. Ch.
  • Soit $a\in \mathbb{R}$, $a\neq 0$. Soit $f_{a}\colon\mathbb{R}\longrightarrow\mathbb{R}$ et $g_{a}\colon\mathbb{R}\longrightarrow\mathbb{R}$ les fonctions définies par
    \begin{equation*}
    f_{a}(x)=
    \begin{cases}
    x\sin\frac{a}{x}& \text{si $x\neq0$,} \\
    0 & \text{si $x=0$}
    \end{cases}
    \qquad\text{et}\qquad
    g_{a}(x)=
    \begin{cases}
    x^{2}\sin\frac{a}{x}& \text{si $x\neq0$,} \\
    0 & \text{si $x=0$.}
    \end{cases}
    \end{equation*}
    La fonction $f_{a}$ est continue sur $\mathbb{R}$, donc elle est la dérivée d'une certaine fonction. La fonction $g_{a}$ est continue et dérivable sur $\mathbb{R}$ et
    \begin{equation*}
    g'_{a}(x)=
    \begin{cases}
    2x\sin\frac{a}{x}-a\cos\frac{a}{x}& \text{si $x\neq0$,} \\
    0 & \text{si $x=0$.}
    \end{cases}
    \end{equation*}
    La fonction $u_{a}\colon\mathbb{R}\longrightarrow\mathbb{R}$ définie par
    \begin{equation*}
    u_{a}(x)=
    \begin{cases}
    \cos\frac{a}{x}& \text{si $x\neq0$,} \\
    0 & \text{si $x=0$}
    \end{cases}
    \end{equation*}
    est donc une dérivée, puisque $u_{a}=\tfrac{1}{a}(2f_{a}-g'_{a})$. De même, la fonction $v_{a}\colon\mathbb{R}\longrightarrow\mathbb{R}$ définie par
    \begin{equation*}
    v_{a}(x)=
    \begin{cases}
    \sin\frac{a}{x}& \text{si $x\neq0$,} \\
    0 & \text{si $x=0$}
    \end{cases}
    \end{equation*}
    est une dérivée. On considère maintenant la fonction $w_{a}\colon\mathbb{R}\longrightarrow\mathbb{R}$ définie par $w_{a}=u_{a}^{2}+v_{a}^{2}$. On a
    \begin{equation*}
    w_{a}(x)=
    \begin{cases}
    1& \text{si $x\neq0$,} \\
    0 & \text{si $x=0$.}
    \end{cases}
    \end{equation*}
    La fonction $w_{a}$ ne vérifie pas la propriété des valeurs intermédiaires donc, d'après le théorème de Darboux, ce n'est pas une dérivée. Il s'ensuit qu'au moins une des fonctions $u_{a}^{2}$ et $v_{a}^{2}$ n'est pas une dérivée. On remarque que $u_{a}^{2}=\tfrac{w_{a}}{2}+\tfrac{u_{2a}}{2}$. Puisque $u_{2a}$ est une dérivée et $w_{a}$ n'en est pas une, on conclut que la fonction $u_{a}^{2}$ n'est pas une dérivée. De même, la fonction $v_{a}^{2}$ n'est pas une dérivée. On a ainsi obtenu deux exemples de fonctions dérivées (les fonctions $u_{a}$ et $v_{a}$) dont le carré n'est pas une dérivée. On note aussi que les fonctions $u_{a}^{2}$ et $v_{a}^{2}$ vérifient la propriété des valeurs intermédiaires, mais que leur somme ne vérifie pas cette propriété.
  • Merci Eric.
    Voir aussi ici :
    http://www.les-mathematiques.net/phorum/read.php?4,1438348,1438704#msg-1438704
    http://www.les-mathematiques.net/phorum/read.php?4,1495256,1495342#msg-1495342

    Et aussi :
    https://www.encyclopediaofmath.org/index.php/Darboux_property

    J'ai posé à nouveau la question pour la remettre au goût du jour et voir s'il n'y avait pas d'autre contre-exemple.

    On notera que les réciproques sont fausses : une fonction de Darboux n'est pas nécessairement continue, et n'est pas nécessairement une fonction dérivée.
    Et le produit de deux fonctions dérivées n'est pas nécessairement une fonction dérivée.

    Bonne nuit.
    Fr. Ch.
  • Le pdf joint (bonne source d'exercices) donne une possibilité de construire à volonté des fonctions dérivées dont le produit n'est pas une fonction dérivée.
  • La démonstration proposée par rakam est intéressante mais relève de l'astuce et a donc un intérêt pédagogique limité. En revanche la démonstration qui consiste par translation à se ramener à une situation plus simple, bref celle du livre, est un raisonnement classique en mathématique. C'est la même idée pour démontrer le TAF à partir de Rolle pour rester dans cette thématique.

    Pour la petite histoire un collègue avait fait un test avec des L1 demandant lors d'une interrogation, les quelques rares étudiants étant parvenus à une ébauche de solution avaient eu la démarche du livre, au moins en tentative, et non les fonctions auxiliaires. C'est dire si elle est effectivement plus naturelle... J ai également remarqué que les candidats qui exposent lors de concours la démonstration de rakam se plantent souvent, jamais ceux qui choisissent la version simple.
  • @Amathoué
    Oui j'espère bien. Les mathématiques du supérieur sont une discipline exigeante et difficile en effet. Il faut être patient. Mon niveau reste modeste dans les raisonnements mais j'espère progresser. Il y a qu'en calcul pur où je pense avoir un bon niveau.

    @Eric
    Merci pour la démonstration en anglais est très bien faite. Je l'ai comprise bien que le théorème soit légèrement différent. Par contre, l'auteur donne l'existence d'un $x \in [a,s]$ alors que dans le théorème des accroissements finis l'intervalle d'existence est ouvert, c'est le seul détail que je n'ai pas compris.
  • Pour compléter toutes ces considérations sur le théorème de Darboux on peut donner un exemple de fonction dérivable à dérivée non continue :
    la fonction $x\mapsto x^2\sin\frac{1}{x}$ prolongée par continuité en 0.
  • @OShine
    je vois que tu comprends mieux l'anglais que le chinois : j'y veillerai pour l'avenir !

    Quant à ton étonnement sur $x\in[a,s]$ il est dû à ton refus de voir (et ce n'est pas la première fois) que l'intervalle ouvert est inclus dans le segment.
  • @Rakam
    C'est juste que vos inégalités sont étranges et je n'ai pas compris le rapport avec le théorème des valeurs intermédiaires ni le théorème de Rolle. Je ne vois pas le lien entre vos inégalités et les théorèmes du cours.

    Dans le document en anglais la démonstration explicite plus clairement les choses et on voit apparaître naturellement les hypothèses des théorèmes.

    Je rappelle que j'ai un niveau très modeste (je peux vite me décourager) et que si les démonstrations ne collent pas à des références explicites du cours je suis vite perdu et je ne comprends absolument rien.

    Cordialement.
  • @celine: avec ton exemple, a-t-on le droit d'écrire $f'(0)=0$ ? Je suppose que non...?
  • Si ! $\frac {f(x)-f(0)}{x-0} = x \sin(\frac1x )$ tend vers 0 quand $x$ tend vers 0.
  • Oui et c'est ce qui fait que $f'$ n'est pas continue car $f'$ définie sur $\mathbb{R}^*$ n'a pas de limite en $0$.
  • Mais je croyais que dérivable $=>$ continue (mais réciproque fausse)...?

    Exemple classique : $\sqrt{x}$ en $0$.
  • Si f est dérivable, elle est continue. Mais la dérivée, elle, n'est pas nécessairement continue.
  • @Yale : ah oui bien sûr merci.
    Elle est drôle cette fonction...
Connectez-vous ou Inscrivez-vous pour répondre.